Line 42: Line 42:
 
d) Do the function&nbsp;<span class="texhtml">''p''<sub>0</sub>(''n'')</span>&nbsp;and&nbsp;<span class="texhtml">''p''<sub>1</sub>(''m'')</span>&nbsp;together contains sufficient information to reconstruction the function&nbsp;<span class="texhtml">''x''(''m'',''n'')</span>? If so, provide a reconstruction algorithm; if not, provide a counter example.  
 
d) Do the function&nbsp;<span class="texhtml">''p''<sub>0</sub>(''n'')</span>&nbsp;and&nbsp;<span class="texhtml">''p''<sub>1</sub>(''m'')</span>&nbsp;together contains sufficient information to reconstruction the function&nbsp;<span class="texhtml">''x''(''m'',''n'')</span>? If so, provide a reconstruction algorithm; if not, provide a counter example.  
  
Click [[QE637 2013 Pro1|here]] to view student [[QE637 2013 Pro1|answers and discussions]] <br>
+
Click [[QE637 2013 Pro1|here]] to view student [[QE637 2013 Pro1|answers and discussions]] <br>  
<br>
+
 
'''Problem 2. ''' (50 pts)  
+
----
 +
 
 +
<br> '''Problem 2. ''' (50 pts)  
  
 
Let <span class="texhtml">''r''<sub>0</sub>(λ)</span>, <span class="texhtml">''g''<sub>0</sub>(λ)</span>, and <span class="texhtml">''b''<sub>0</sub>(λ)</span> be the CIE color matching functions for red, green, and blue primaries at 700 nm, 546.1 nm, and 435.8 nm, respectively, and let <span class="texhtml">[''r'',''g'',''b'']</span>&nbsp;be the corresponding CIE tristimulus values.&nbsp;&lt;/span&gt;  
 
Let <span class="texhtml">''r''<sub>0</sub>(λ)</span>, <span class="texhtml">''g''<sub>0</sub>(λ)</span>, and <span class="texhtml">''b''<sub>0</sub>(λ)</span> be the CIE color matching functions for red, green, and blue primaries at 700 nm, 546.1 nm, and 435.8 nm, respectively, and let <span class="texhtml">[''r'',''g'',''b'']</span>&nbsp;be the corresponding CIE tristimulus values.&nbsp;&lt;/span&gt;  
Line 83: Line 85:
  
 
Click [[QE637 2014 Pro2|here]] to view student [[QE637 2014 Pro2|answers and discussions]]  
 
Click [[QE637 2014 Pro2|here]] to view student [[QE637 2014 Pro2|answers and discussions]]  
 
  
 
[[Category:ECE]] [[Category:QE]] [[Category:CNSIP]] [[Category:Problem_solving]] [[Category:Image_processing]]
 
[[Category:ECE]] [[Category:QE]] [[Category:CNSIP]] [[Category:Problem_solving]] [[Category:Image_processing]]

Revision as of 16:57, 11 November 2014


ECE Ph.D. Qualifying Exam

Communication, Networking, Signal and Image Processing (CS)

Question 5: Image Processing

August 2013



Question

Question is posted from this link.

Problem 1. (50 pts)

Consider the 2D discrete space signal x(m,n) with the DSFT of X(ejμ,ejν) given by 

$ X(e^{j\mu},e^{j\nu}) = \sum_{m=-\infty}^{\infty} \sum_{n=-\infty}^{\infty} x(m,n)e^{-j(m\mu+n\nu)} $

Then define

$ p_{0}(n) = \sum_{m=-\infty}^{\infty}x(m,n) $

$ p_{1}(m) = \sum_{n=-\infty}^{\infty}x(m,n) $

with corresponding DTFT given by 

$ P_{0}(e^{j\omega}) = \sum_{n=-\infty}^{\infty} p_{0}(n)e^{-jn\omega} $

$ P_{1}(e^{j\omega}) = \sum_{m=-\infty}^{\infty} p_{0}(m)e^{-jm\omega} $
a) Derive an expression for P0(ejω) in terms of X(ejμ,wjν).

b) Derive an expression P0(ejω) in terms of X(ejμ,ejν).

c) Derive an expression  for $ \sum_{n = -\infty}^{\infty}p_0(n) $ interms of X(ejμ,ejν).

d) Do the function p0(n) and p1(m) together contains sufficient information to reconstruction the function x(m,n)? If so, provide a reconstruction algorithm; if not, provide a counter example.

Click here to view student answers and discussions



Problem 2. (50 pts)

Let r0(λ), g0(λ), and b0(λ) be the CIE color matching functions for red, green, and blue primaries at 700 nm, 546.1 nm, and 435.8 nm, respectively, and let [r,g,b] be the corresponding CIE tristimulus values. </span>

Furthermore, let f1(λ)f2(λ), and f3(λ) be the spectral response functions for the three color outputs of a color camera. So for each pixel of the camera sensor, there is a 3-dimensional output vector given by F = [F1,F2,F3]t, where

$ F_1 = \int_{-\infty}^{\infty}f_1(\lambda)I(\lambda)d\lambda $,

$ F_2 = \int_{-\infty}^{\infty}f_2(\lambda)I(\lambda)d\lambda $,

$ F_3 = \int_{-\infty}^{\infty}f_3(\lambda)I(\lambda)d\lambda $

where I(λ) is the energy spectrum of the incoming light and $ f_k(\lambda)\geq 0 $ for k = 0,1,2..

Furthermore, assume there exists a matrix, M, so that

$ \left[ {\begin{array}{*{20}{c}} f_1(\lambda)\\ f_1(\lambda)\\ f_1(\lambda) \end{array}} \right] = {\begin{array}{*{20}{c}} M \end{array}} \left[ {\begin{array}{*{20}{c}} r_0(\lambda)\\ g_0(\lambda)\\ b_0(\lambda) \end{array}} \right] $


a) Why is it necessary that $ f_k(\lambda) \geq 0 $ for k = 0,1,2?

b) Are the functions, $ r_0(\lambda) \geq 0 $, $ g_0(\lambda) \geq 0 $, and $ b_0(\lambda) \geq 0 $? If so, why? If not, why not?

c) Derive an formula for the tristimulus vector [r,g,b]t in terms of the tristimulus vector F = [F1,F2,F3]t.

d) Do functions fk(λ) exist, which meet these requirements? If so, give a specific example of such functions.

Click here to view student answers and discussions

Alumni Liaison

Meet a recent graduate heading to Sweden for a Postdoctorate.

Christine Berkesch